5
$\begingroup$

Let $M(n)$ be an $n\times n$ matrix in the variables $x_1,\dots,x_n$ with entries $$M_{i,j}(n)=\frac{x_{\max(i,j)}}{x_{\min(i,j)}}, \qquad 1\leq i,j\leq n.$$ I'm interested in the following:

Questions.

(1) Is there a neat or "closed form" evaluation for the determinant $\det M(n)$?

(2) Is there an explicit formula for the inverse of $M(n)$?

Thank you.

$\endgroup$

2 Answers 2

10
$\begingroup$

Let us write $$a_r=\frac{x_{r+1}}{x_r}$$ for $r=1\cdots n$.

We can then write the matrix $M(n)$ in the form $$\begin{pmatrix} 1 & a_1 & a_1a_2& \cdots & a_1a_2\cdots a_{n-1} \\ a_1 & 1 & a_2& \cdots & a_2\cdots a_{n-1}\\ \vdots & \vdots & \vdots &\ddots & \vdots\\ a_1a_2\cdots a_{n-1}& a_2\cdots a_{n-1} &\cdots & a_{n-1} & 1\end{pmatrix} $$ We do now Gauss elimination, and reduce $a_{n-1}$ times the $(n-1)$-th row from the $n$-th row. We then get in the last row $0, 0, \ldots (1-a_{n-1}^2)$. But this means that $det(M(n))=det(M(n-1))(1-a_{n-1}^2)$ and by induction $$det(M(n)) = \prod_{r=1}^{n-1} (1-a_r^2)=\prod_{r=1}^{n-1}(1-\frac{x_{r+1}^2}{x_r^2}).$$ By using inductively the Gauss elimination mentioned above, one can also get the inverse of $M(n)$.

$\endgroup$
7
  • $\begingroup$ Thanks. What is the inverse, explicitly? $\endgroup$ Jan 4, 2017 at 16:04
  • 1
    $\begingroup$ Nice. The last product should also contain squares, right? $\endgroup$
    – Dirk
    Jan 4, 2017 at 16:09
  • $\begingroup$ I didn't do the exact calculations for the inverse matrix. What I can say is the following: We can use Cramer's Rule. Then all the $n-1\times n-1$ minors will either have zero determinant, or will be something of the form $a_i \cdot M(n-1)$ for some $i$ (and $M(n-1)$ stands here for some reordering of the indices). You will then get quite an explicit formula. $\endgroup$
    – Ehud Meir
    Jan 4, 2017 at 16:45
  • 2
    $\begingroup$ Regarding $a_i$ as an indeterminate in the matrix of Ehud's answer, the matrix is the Varchenko matrix of the arrangement of $n-1$ parallel lines in the plane. See for instance Section 6.5 of cis.upenn.edu/~cis610/sp06stanley.pdf. Theorem 6.25 of this reference therefore gives a vast generalization of $\det M(n)$. The inverse of $M(n)$ is a tridiagonal matrix with terms $a_i/(a_i^2-1)$ on the diagonal above and below the main diagonal, and with $(1,1)$-entry $1/(1-a_1^2)$, $(n,n)$-entry $1/(1-a_{n-1}^2)$, and other diagonal entries $(1-a_i^2a_{i+1}^2)/(1-a_i^2)(1-a_{i+1}^2)$. $\endgroup$ Jan 4, 2017 at 17:37
  • 2
    $\begingroup$ A slick way to evaluate Ehud's matrix is notice that two rows become equal if we set $a_i=1$ and become negatives if we set $a_i=-1$. A simple degree argument shows that there are no other factors, and setting each $a_i=0$ gives the correct scaling factor (i.e., the polynomial has constant term 1). $\endgroup$ Jan 4, 2017 at 22:21
3
$\begingroup$

I wish to advertise the Method of Condensation in proving determinantal evaluation. The key is guess the answer, which is thanks to Ehud Meir. Then, generalize it a bit. Let $x_1, x_2,\dots$ be an infinite set of variables and modify the original matrix (by shifting variables) to $M^{a,b}(n)$ so that $$M_{i,j}^{a,b}(n):=\frac{x_{\max(i+a,j+b)}}{x_{\min(i+a,j+b)}}.$$ Convention: $M^{0,0}(n)=M(n)$.

Claim. If $a\neq b$ then $\det M^{a,b}(n)=0$, and if $a=b$ then $$\det M^{a,a}(n)=\prod_{r=2}^n\frac{x_{k-1+a}^2-x_{k+a}^2}{x_{k-1+a}^2}.\tag1$$ Proof. The case $a\neq b$ is easy - simply factor out a variable from $n^{th}$-column/row and another variable from the $(n-1)^{th}$-column/row. These new columns/rows are identical.

Inductive proofs neatly work with this Dodgson's recursive relation $$\det Z^{0,0}(n)=\frac{\det Z^{1,1}(n-1)\det Z^{1,1}(n-1)-\det Z^{0,1}(n-1)\det Z^{1,0}(n-1)}{\det Z^{1,1}(n-2)}$$ satisfied by any matrix (so long as denominators do not vanish). Thus, it holds for $\det M^{a,b}(n)$. So, it remains to prove that the (explicit) formula on the RHS of (1) does satisfy the same equation. However, this is quite a routine simplification (preferably with symbolic sofwares). The proof follows. $\square$

$\endgroup$
2

Your Answer

By clicking “Post Your Answer”, you agree to our terms of service and acknowledge that you have read and understand our privacy policy and code of conduct.

Not the answer you're looking for? Browse other questions tagged or ask your own question.